2
$\begingroup$

For definitions, see Section 1 of Chapter 3 of Richard Stanley, Enumerative Combinatorics, Volume I (second edition). Also see Section 8 of Chapter II of Garrett Birkhoff, Lattice Theory (third edition).

A poset is semimodular if whenever an element x is covered by distinct elements y and z, there exists an element w covering y and z. It is known that finite semimodular posets with a least element are ranked.

Can you find a finite (ranked) semimodular poset with ranks 0 through 2 looking like the figure below, with the additional property that every rank 2 interval has either 3 or 4 elements (i.e., is a 3-element chain or a diamond) except for the interval [0,d]?

enter image description here

Robert A. Proctor and Lindsey M. Scoppetta, "d-Complete Posets: Local Structural Axioms, Properties, and Equivalent Definitions," Order 36 (2019), 399-422.

$\endgroup$
2
  • $\begingroup$ Not a critique of your question or anything, but I've never heard of "semimodular" used outside the context of lattices. $\endgroup$ Commented Apr 2, 2024 at 13:22
  • 1
    $\begingroup$ @SamHopkins: see for instance ams.org/journals/proc/1971-028-02/S0002-9939-1971-0276144-6/… $\endgroup$ Commented Apr 2, 2024 at 14:30

1 Answer 1

4
$\begingroup$

In GraphViz notation,

digraph {
    rankdir="BT";
    edge [arrowhead=none];
    0 -> a; 0 -> b; 0 -> c;
    a -> ab; a -> ac; a -> d;
    b -> ab; b -> bc; b -> d;
    c -> ac; c -> bc; c -> d;
    ab -> D; ab -> AB;
    ac -> D; ac -> AC;
    d -> AB; d -> AC; d -> BC;
    bc -> D; bc -> BC;
    D -> A; D -> B;
    AB -> A; AB -> B;
    AC -> A; AC -> C;
    BC -> B; BC -> C;
    A -> 1;
    B -> 1;
    C -> 1;
}

Rendered by dot

It has a pleasing symmetry (with one tiny exception, where $C$ doesn't cover $D$) which I've tried to bring out with the labels.

$\endgroup$
3
  • 1
    $\begingroup$ This is a beautiful poset, but the interval $[D, 1]$ has five elements. I believe it can be fixed by adding one more level. $\endgroup$ Commented Apr 3, 2024 at 12:32
  • 1
    $\begingroup$ @JukkaKohonen, I missed that when trying to increase the symmetry. The fix is simply to make $C$ not cover $D$. $\endgroup$ Commented Apr 3, 2024 at 13:33
  • $\begingroup$ Thank you. This appears to resolve Conjecture 4.3 (negatively) in the above paper of Proctor and Scoppetta. $\endgroup$ Commented Apr 4, 2024 at 4:03

You must log in to answer this question.

Start asking to get answers

Find the answer to your question by asking.

Ask question

Explore related questions

See similar questions with these tags.